Download as pdf or txt
Download as pdf or txt
You are on page 1of 57

ಗೆಲುವು GROUP C TEST SERIES

Q1) The distribution of powers between the Centre and the States in the Indian Constitution is
based on the scheme provided in which of the following acts?
A. Morley-Minto Reforms, 1909
B. Montagu-Chelmsford Act, 1919
C. Government of India Act, 1935
D. Indian Independence Act, 1947

Ans: C
Government of India Act of 1935
• It provided for the establishment of an All India federation consisting of provinces and
princely states as units.
• It divided the powers between the centre and units in terms of three lists- Federal list,
provincial list and the concurrent list. Residuary powers were given to the Viceroy.
However, this federation never fructified since princely states did not join it.
• It abolished dyarchy in the provinces and introduced ‘provincial autonomy’ in its place
• The act introduced responsible government in provinces, that is, the governor was required
to act with the advice of ministers responsible to the provincial legislature
• It provided for the adoption of dyarchy at the centre. However, this provision did not come
into effect at all
• Bicameralism was introduced in six provinces- Bengal, Bombay Madras, Bihar, Assam
and the United Provinces
• Separate electorates was further extended to depressed classes, women and labour
• Council of India which was established as per the 1858 act was abolished The secretary
of state was instead provided with a team of advisors.
• The act provided for setting up- Federal public service commission, provincial public
service commission, joint public service commission, federal court, Reserve Bank of India

Q2) Which of the following are features of the Indian Parliamentary System:
1. Independent Judiciary
2. Collective responsibility of the executive to the legislature
3. A written Constitution
4. Presence of de jure and de facto executives
5. Individual responsibility of the executive to the legislature.
Choose the correct option
A. 2,3 and 4
B. 1,2 and 4
C. 2, 4 and 5
D. 1,2,4 and 5

Ans: C

1
©Tathagat GS-Prep Ph: 8431824230/9113994724
ಗೆಲುವು GROUP C TEST SERIES

Features of Parliamentary Government


• Nominal and Real Executives: The President is the nominal executive (also known as the
de jure or titular executive), whereas the Prime Minister is the real executive (de facto
executive). As a result, the President is the President of the State, whereas the Prime
Minister is the Prime Minister of the Government.
• Double Membership: The Prime Minister and the Council of Ministers serve as the
executive, while the Parliament serves as the legislature. Members of parliament elect the
Prime Minister and ministers, meaning that the executive comes from the legislative.
• Collective Responsibility: The executive has a collective responsibility to the legislative.
There is a collective responsibility, which means that each minister's responsibilities are
shared by the entire Council.
• Secrecy of procedure: A requirement of this type of administration is that cabinet
proceedings be kept secret and not made public.
• Prime Ministerial Leadership: The Prime Minister is in charge of this system of
administration.
• Majority Party Rule: The Prime Minister is usually appointed by the leader of the party
that obtains a majority in the lower chamber.
• Bicameral Legislature: Bicameral legislatures are used in most parliamentary
democracies.
• Political Homogeneity: Members of the council of ministers are usually from the same
political party, and so have similar political ideologies. The ministers in a coalition
government are bound by consensus.
• No fixed term: The government's term is determined by the lower house's majority
support. The council of ministers must resign if the government fails to win a vote of no
confidence. There will be elections, and a new government will be formed.

Merits of Parliamentary System


• Coordination between the legislature and the executive
• Responsible Government
• Prevents Despotism
• Alternative Government
• Wide Representation

Q3) Through which of the following acts the foundation of Central Administration was laid in
India?
A. Charter Act of 1833
B. Regulating Act of 1773
C. Charter Act of 1853
D. Pitt’s India Act of 1784

2
©Tathagat GS-Prep Ph: 8431824230/9113994724
ಗೆಲುವು GROUP C TEST SERIES

Ans: B
Regulating Act of 1773: This act was of great constitutional importance as
• it was the first step taken by the British Government to control and regulate the affairs of
the East India Company in India;
• it recognised, for the first time, the political and administrative functions of the Company;
and
• it laid the foundations of central administration in India.
The Charter Act of 1833 was considered as the final step towards centralization in British India.

Q4) The electoral college of the President of India consists of which of the following:
A. Elected and nominated Members of Parliament and State legislatures
B. Elected members of Parliament and state legislatures
C. Elected members of Parliament but not the elected and nominated in case of a bicameral
legislature.
D. Elected members of all the Union territories.

Ans: B
Article 54 of the constitution says: "The President shall be elected by the members of an electoral
college consisting of:
(a) The elected members of both Houses of Parliament and
(b) The elected members of the Legislative Assemblies of the States (including National Capital
Territory of Delhi and the Union Territory of Pondicherry vide the Constitution 70th amendment
Act, 1992)."
How is the President elected?
▪ About:
o The Indian President is elected through an electoral college system, wherein the
votes are cast by national and State-level lawmakers.
o The elections are conducted and overseen by the Election Commission (EC) of
India.
• The electoral college is made up of all the elected members of the Upper
and Lower Houses of Parliament (Rajya Sabha and Lok Sabha MPs), and
the elected Members of the Legislative Assemblies of States and Union
Territories (MLAs).
o Related Constitutional Provisions:
• Article 54: Election of President
• Article 55: Manner of election of President.
• Article 56: Term of office of President
• Article 57: Eligibility for re-election.
• Article 58: Qualifications for election as President

3
©Tathagat GS-Prep Ph: 8431824230/9113994724
ಗೆಲುವು GROUP C TEST SERIES

▪ Procedure:
o Before the voting, comes the nomination stage, where the candidate intending to
stand in the election, files the nomination along with a signed list of 50 proposers
and 50 seconders.
o These proposers and seconders can be anyone from the total members of the
electoral college from the State and national level.
• The rule for securing 50 proposers and seconders was implemented
when the EC noticed, in 1974, that several candidates, many without even
a bleak chance of winning, would file their nominations to contest the polls.
o An elector cannot propose or second the nomination of more than one
candidate.

Q5) Prime Minister does not chair Which of the following Cabinet Committees?
A. Political Affairs Committee
B. Appointments Committee
C. Committee on Parliamentary Affairs
D. Economic Affairs Committee

Ans: A
Eight Cabinet Committees:

4
©Tathagat GS-Prep Ph: 8431824230/9113994724
ಗೆಲುವು GROUP C TEST SERIES

• Appointments Committee of the Cabinet.


• Cabinet Committee on Accommodation.
• Cabinet Committee on Economic Affairs.
• Cabinet Committee on Parliamentary Affairs.
• Cabinet Committee on Political Affairs.
• Cabinet Committee on Security.
• Cabinet Committee on Investment and Growth.
• Cabinet Committee on Employment & Skill Development.
o All committees except Cabinet Committee on Accommodation and Cabinet Committee on
Parliamentary Affairs are headed by the Prime Minister.
o They are extra-constitutional in emergence.
o In other words, they are not mentioned in the Constitution. However, the Rules of
Business provide for their establishment.
o The executive in India works under the Government of India Transaction of Business
Rules,1961.
• These Rules emerge from Article 77(3) of the Constitution, which states: “The
President shall make rules for the more convenient transaction of the business of
the Government of India, and for the allocation among Ministers of the said
business.”
o The Prime Minister constitutes Standing Committees of the Cabinet and sets out
the specific functions assigned to them. He can add or reduce the number of committees.
• In addition to cabinet committees, several Groups of Ministers (GoMs) are
constituted to look into different issues/subjects.

Q6) The President can perform what function out of the following in the declaration of
constitutional emergency?
1. Assume to himself all the functions of the state government, including the high court
2. Declare that the powers of the state legislature shall be exercisable under the authority of
the governor
3. Assume to himself all the functions of the state government except the High Court
4. Declare that the powers of the state legislature shall be exercisable under the authority of
the Parliament.
Of the above, the correct statements are:
A. 1and 2
B. 2 and 3
C. 3 and 4
D. 1 and 4

Ans: C
▪ The Constitution stipulates three types of emergencies-

5
©Tathagat GS-Prep Ph: 8431824230/9113994724
ಗೆಲುವು GROUP C TEST SERIES

1. National Emergency
2. Constitutional Emergency
3. Financial Emergency
NATIONAL EMERGENCY
▪ National emergency can be declared on the basis of war, external aggression or armed
rebellion. The Constitution employs the expression ‘proclamation of emergency’ to denote
an emergency of this type.
▪ The 38th Amendment Act of 1975 made the declaration of National Emergency immune
to judicial review. But, this provision was subsequently deleted by the 44th Amendment
Act of 1978.
▪ In Minerva Mills case (1980), the Supreme Court held that National Emergency can be
challenged in the court on the ground of malafide or that the declaration was based on
wholly extraneous and irrelevant facts.

CONSTITUTIONAL EMERGENCY or PRESIDENT’S RULE


▪ Consequences of the President’s rule: The President acquires the following
extraordinary powers when the President’s rule is imposed in a state:

o He can take up the functions of the state government and powers vested in the
governor or any other executive authority in the state.
o He can declare that the powers of the state legislature are to be exercised by the
parliament.
o He can take all other necessary steps including the suspension of the constitutional
provisions relating to anybody or authority in the state.
▪ Scope of judicial review: The 38th Amendment act of 1975 made the satisfaction of the
President in invoking Article 356 final and conclusive which would not be challenged in
any court on any ground.
o But, this provision was subsequently deleted by the 44th Amendment Act of 1978
implying that the satisfaction of the President is not beyond judicial review.

Q7) Which of the following is not a formally prescribed device available to the members of
Parliament?
A. Question Hour
B. Zero Hour
C. Half- an- hour discussion
D. Short duration discussion

Ans: B

List of Devices of Parliamentary Proceedings

6
©Tathagat GS-Prep Ph: 8431824230/9113994724
ಗೆಲುವು GROUP C TEST SERIES

The table below mentions the different devices of Parliamentary Proceedings:


Devices of Facts about Parliamentary Proceedings Devices
Parliamentary
Proceedings

Question Hour The first hour of every parliamentary sitting is slotted for this. During this
time, the members ask questions and the ministers usually give answers. The
questions are of three kinds, namely, starred, unstarred and short notice.
1. A starred question (distinguished by an asterisk) requires an oral answer
and hence supplementary questions can follow.
2. An unstarred question, on the other hand, requires a written answer and
hence, supplementary questions cannot follow.
3. A short notice question is one that is asked by giving a notice of less than
ten days. It is answered orally.

Closure Motion It is a motion moved by a member to cut short the debate on a matter before
the House. If the motion is approved by the House, debate is stopped forthwith
and the matter is put to vote. There are four kinds of closure motions:
(a) Simple Closure: It is one when a member moves that the ‘matter having
been sufficiently discussed be now put to vote’.
(b) Closure by Compartments: In this case, the clauses of a bill or a lengthy
resolution are grouped into parts before the commencement of the debate. The
debate covers the part as a whole and the entire part is put to vote.
(c) Kangaroo Closure: Under this type, only important clauses are taken up
for debate and voting and the intervening clauses are skipped over and taken
as passed.
(d) Guillotine Closure: It is one when the undiscussed clauses of a bill or a
resolution are also put to vote along with the discussed ones due to want of
time (as the time allotted for the discussion is over).

No-Confidence Article 75 of the Constitution says that the council of ministers shall be
Motion collectively responsible to the Lok Sabha. It means that the ministry stays in
office so long as it enjoys confidence of the majority of the members of the
Lok Sabha. In other words, the Lok Sabha can remove the ministry from office
by passing a no-confidence motion. The motion needs the support of 50
members to be admitted.

Privilege Motion It is concerned with the breach of parliamentary privileges by a minister. It is


moved by a member when he feels that a minister has committed a breach of
privilege of the House or one or more of its members by withholding facts of

7
©Tathagat GS-Prep Ph: 8431824230/9113994724
ಗೆಲುವು GROUP C TEST SERIES

a case or by giving wrong or distorted facts. Its purpose is to censure the


concerned minister.

Motion of Thanks The first session after each general election and the first session of every fiscal
year is addressed by the president. In this address, the president outlines the
policies and programmes of the government in the preceding year and ensuing
year. This address of the president, which corresponds to the ‘speech from the
Throne in Britain’, is discussed in both the Houses of Parliament on a motion
called the ‘Motion of Thanks’. At the end of the discussion, the motion is put
to vote. This motion must be passed in the House. Otherwise, it amounts to
the defeat of the government. This inaugural speech of the president is an
occasion available to the members of Parliament to raise discussions and
debates to ex-amine and criticise the government and administration for its
lapses and failures.

Calling Attention It is introduced in the Parliament by a member to call the attention of a


Motion minister to a matter of urgent public importance, and to seek an authoritative
statement from him on that matter. Like the zero hour, it is also an Indian
innovation in the parliamentary procedure and has been in existence since
1954. However, unlike the zero hour, it is mentioned in the Rules of
Procedure.

Censure Motion It should state the reasons for its adoption in the Lok Sabha. It can be moved
against an individual minister or a group of ministers or the entire council of
ministers. It is moved for censuring the council of ministers for specific
policies and actions. If it is passed in the Lok Sabha, the council of ministers
need not resign from the office.

Half-an-Hour It is meant for discussing a matter of sufficient public importance, which has
Discussion been subjected to a lot of debate and the answer to which needs elucidation on
a matter of fact. The Speaker can allot three days in a week for such
discussions. There is no formal motion or voting before the House.

Short Discussion It is also known as two-hour discussion as the time allotted for such a
discussion should not exceed two hours. The members of the Parliament can
raise such discussions on a matter of urgent public importance. The Speaker
can allot two days in a week for such discussions. There is neither a formal
motion before the house nor voting. This device has been in existence since
1953.

8
©Tathagat GS-Prep Ph: 8431824230/9113994724
ಗೆಲುವು GROUP C TEST SERIES

Adjournment When there is an urgent matter of public importance then a member may
Motion propose that the business of the house be adjourned for discussing that matter.
This motion can be moved only with the consent of the Speaker. Generally,
such motions are discussed in the afternoon at 4.00 p.m.

Lame Duck Session It refers to the last session of the existing Lok Sabha, after a new Lok Sabha
has been elected. Those members of the existing Lok Sabha who could not get
re-elected to the new Lok Sabha are called lame-ducks.

Q8) Which of the following is exclusively a committee of the Lower House?


A. Committee on Assurances
B. Committee on Delegated Legislation
C. Committee on Public Undertakings
D. Estimates Committee

Ans: D
Types of Parliamentary Committees
• Based on purpose and duration.
1. Adhoc.
2. Standing – Advisory and Enquiry.
• Based on composition.
1. Select – Single House, ie either LS or RS.
2. Joint – Both Houses.
-Committee Types- Standing Committee Adhoc Committee

Select Committee Eg: Estimate (LS), Ethics Committee (RS) Committees on Bills (Select)

Joint Committee Eg: Public Account Committee Committees on Bills (Joint)

Q9) Which state created the institution of Lokayukta for the first time?
A. Orissa
B. Bihar
C. Punjab
D. Maharashtra

Ans: D
Maharashtra was the first state to introduce Lokayukta through The Maharashtra Lokayukta and
Upa-Lokayuktas Act in 1971.

9
©Tathagat GS-Prep Ph: 8431824230/9113994724
ಗೆಲುವು GROUP C TEST SERIES

o The Lokayukta is the Indian Parliamentary Ombudsman, executed into


power, through and for, each of the State Governments of India.
o It is an anti-corruption authority. The object of Lokayukta system in a state is to
make investigation of grievances, allegations against public servants.
▪ Origin:
o The origin of the Lokayukta can be drawn to the Ombudsman in Scandinavian
countries.
o In India, the Administrative Reforms Commission, (1966-70), had recommended
the creation of the Lokpal at the Centre and Lokayukta in the states.
o Before the passing of the Lokpal and Lokayuktas Act in 2013, several states in India
passed laws for creating the Institution of 'Lokayukta'.
• Maharashtra was first in this respect with its Lokayukta body established
in 1971.
▪ Appointment:
o The lokayukta and upa-lokayukta are appointed by the Governor of the state.
While appointing, the governor in most of the states consults (a) the chief justice
of the state high court, and (b) the leader of Opposition in the state legislative
assembly.
▪ Tenure:
o In most of the states, the term of office fixed for lokayukta is of 5 years duration
or 65 years of age, whichever is earlier. He is not eligible for reappointment for
a second term.

Q10. On the recommendation of which of the following committee was the Central Vigilance
Commission set up?
A. First Administrative Reform Commission of India
B. Gorwala Report
C. Kripalani Committee
D. Santhanam Committee

Ans: D
Committee on Prevention of Corruption:
It was set up by the Government of India Resolution on 11 February 1964, on the recommendations
of the Committee on Prevention of Corruption, headed by K. Santhanam, to advise and guide
Central Government agencies in the field of vigilance.

Recently, the Government appointed Suresh N Patel as the new chief of the Central Vigilance
Commission.
What is the Central Vigilance Commission (CVC)?
▪ About:

10
©Tathagat GS-Prep Ph: 8431824230/9113994724
ಗೆಲುವು GROUP C TEST SERIES

o The Central Vigilance Commission was set up by the Government in 1964 on the
recommendations of the Committee on Prevention of Corruption, headed by
Shri K. Santhanam, to advise and guide Central Government agencies in the field
of vigilance.
o CVC are conceived to be the apex vigilance institution, free of control from any
executive authority, monitoring all vigilance activity under the Central
Government and advising various authorities in Central Government organizations
in planning, executing, reviewing and reforming their vigilant work.
o The Parliament enacted Central Vigilance Commission Act, 2003 (CVC Act)
conferring statutory status on the CVC.
o It is an independent body which is only responsible to Parliament.
o It submits its report to the President of India.
▪ Members:
o Central Vigilance Commissioner - Chairperson.
o Not more than two Vigilance Commissioners - Members.
▪ Appointment:
o The Central Vigilance Commissioner is to be appointed by the President of
India after the recommendation of a three-member committee which consist,
▪ Prime Minister
▪ Minister of Home Affairs
▪ Leader of Opposition in Lok Sabha
▪ Term: holds the office for 4 years.
▪ Removal:
o Can be removed or suspended from the office by the President on the ground
of misbehavior but only after the Supreme Court has held an inquiry into his
case and recommended action against him.
o Further, He can also be removed for proved misbehavior or incapacity if the
Supreme Court inquiry finds him guilty.

Q11. Consider the following statements.


1. Mustard is a rabi crop.
2. Castor seed is grown both as rabi and kharif crop.
3. Sesamum is a kharif crop in north India and a rabi crop in south India.
Select the correct answer using the codes below.
A. 3 only
B. 1, 2 and 3
C. 2 and 3 only
D. 1 and 2 only

11
©Tathagat GS-Prep Ph: 8431824230/9113994724
ಗೆಲುವು GROUP C TEST SERIES

Ans: B
The agricultural crop year in India is from July to June. The Indian cropping season is classified
into two main seasons-
(i) Kharif and
(ii) Rabi based on the monsoon.
• The kharif cropping season is from July –October during the south-west monsoon and the
Rabi cropping season is from October-March (winter).
• The crops grown between March and June are summer crops. Pakistan and Bangladesh are
two other countries that are using the term ‘kharif’ and ‘rabi’ to describe about their
cropping patterns. The terms ‘kharif’ and ‘rabi’ originate from Arabic language where
Kharif means autumn and Rabi means spring.
• The kharif crops include rice, maize, sorghum, pearl millet/bajra, finger millet/ragi
(cereals), arhar (pulses), soyabean, groundnut (oilseeds), cotton etc.
• The rabi crops include wheat, barley, oats (cereals), chickpea/gram (pulses), linseed,
mustard (oilseeds) etc. Sesamum is a kharif crop in north and rabi crop in south India.
Castor seed is grown both as rabi and kharif crop.
• Different oil seeds are grown covering approximately 12 per cent of the total cropped area
of the country. Main oil-seeds produced in India are groundnut, mustard, coconut,
sesamum (til), soyabean, castor seeds, cotton seeds, linseed and sunflower. Most of these
are edible and used as cooking mediums

Q12. Consider the following about the Global Footprint Network, recently seen in news.
1. It is a subsidiary agency of the United Nations.
2. The network helps to measure the amount of resources we use compared to what we have.
3. National Footprint and Biocapacity Accounts is published by the network.
Select the correct answer using the codes below.
A. 1 and 2 only
B. 2 and 3 only
C. 1 only
D. 1, 2 and 3

Ans: B
Global Footprint Network, founded in 2003, is an independent think tank originally based in the
United States, Belgium and Switzerland. It was established as a charitable not-for-profit
organization in each of those three countries. Global Footprint Network develops and promotes
tools for advancing sustainability, including the ecological footprint and biocapacity, which
measure the amount of resources we use and how much we have. These tools aim at bringing
ecological limits to the center of decision-making. Global Footprint Network's goal is to create a
future where all humans can live well, within the means of one planet Earth. The organization is

12
©Tathagat GS-Prep Ph: 8431824230/9113994724
ಗೆಲುವು GROUP C TEST SERIES

headquartered in Oakland, California. The Network brings together over 70 partner organizations,
including WWF International, ICLEI, Bank Sarasin, The Pictet Group, the New Economics
Foundation, Pronatura México, and the Environment Agency Abu Dhabi.

Q13. Which of these Himalayan peaks lie in India?


1. Makalu
2. Kanchenjunga
3. Namcha Barwa
4. Gurla Mandhata
Select the correct answer using the codes below.
A. 1 and 2 only
B. 1, 2 and 4 only
C. 2 and 3 only
D. 1, 3 and 4 only

Ans: C

Q14. Consider the following statements.


Assertion (A): Locating coal processing industries near coal deposits introduces significant cost
cutting on transport for companies.
Reason (R): Coal is a weight losing raw material.
In the context of the above, which of these is correct?
A. A is correct, and R is an appropriate explanation of A.
B. A is correct, but R is not an appropriate explanation of A.
C. A is correct, but R is incorrect.
D. Both A and R are incorrect.

Ans: A
Coal is a bulky material, which loses weight on use as it is reduced to ash. So, it is not profitable
for companies to transport coal to long distances as a lot of coal content is not useful. Locating
near the coalfields saves on such costs. Coal in India is found in Gondawana deposits in the Eastern
belt, and Tertiary belts in North-east

Q15. Consider the following statements.


1. In India, the executive is a part of the legislature.
2. The parliamentary form of government emphasizes on the interdependence between the
legislative and executive organs.
Which of the above is/are correct?
A. 1 only
B. 2 only

13
©Tathagat GS-Prep Ph: 8431824230/9113994724
ಗೆಲುವು GROUP C TEST SERIES

C. Both 1 and 2
D. Neither 1 nor 2

Ans: C
The parliamentary system of government, which is also known as the Cabinet Government, is
based on close relationship between the executive and legislature. The executive is accountable to
the legislature and stays in office inly as long as it enjoys the confidence of legislature. Under
parliamentary system of government there are two types of executives- nominal and real. The real
executive is accountable to the legislature and when later passes a vote of confidence against it has
to render its resignation or seek the dissolution of the legislature. In India, executive is a part of
the legislature, and the legislature is led by the executive. There significant interdependence
between the two and there is no strict separation of powers

Q16. Consider the following statements.


1. More than 20 per cent of the net sown area in the country is organically farmed.
2. Madhya Pradesh, Rajasthan and Maharashtra account for about half the area under organic
cultivation in India.
Select the correct answer using the codes below.
A. 1 only
B. 2 only
C. Both 1 and 2
D. Neither 1 nor 2

Ans: C
Organic farming is in a nascent stage in India. About 2.78 million hectare of farmland was under
organic cultivation as of March 2020, according to the Union Ministry of Agriculture and Farmers’
Welfare. This is two per cent of the 140.1 million ha net sown area in the country.

A few states have taken the lead in improving organic farming coverage, as a major part of this
area is concentrated only in a handful of states. Madhya Pradesh tops the list with 0.76 million ha
of area under organic cultivation — that is over 27 per cent of India’s total organic cultivation
area. The top three states — Madhya Pradesh, Rajasthan and Maharashtra — account for about
half the area under organic cultivation.

The top 10 states account for about 80 per cent of the total area under organic cultivation. Sikkim
is the only Indian state to have become fully organic so far. A majority of the states have only a
small part of their net sown area under organic farming. Even the top three states that account for
the largest area under organic cultivation — Madhya Pradesh, Rajasthan and Maharashtra — have
only around 4.9, 2.0 and 1.6 per cent of their net sown area under organic farming respectively.

14
©Tathagat GS-Prep Ph: 8431824230/9113994724
ಗೆಲುವು GROUP C TEST SERIES

A few states such as Meghalaya, Mizoram, Uttarakhand, Goa and Sikkim have 10 per cent or more
of their net sown area under organic cultivation. All these states, except Goa, are in hilly regions.

Q17. Which of the following drainage pattern develops where the river channel follows the slope
of the terrain?
A. Dendritic
B. Radial
C. Rectangular
D. Trellis

Ans: A
The streams within a drainage basin form various patterns depending on the slope of land,
underlying rock structure as well as the climatic conditions of the area. These are dendritic, trellis,
rectangular, and radial patterns.
• The dendritic pattern develops where the river channel follows the slope of the terrain.
The stream with its tributaries resembles the branches of a tree, thus the name dendritic.
• A river joined by its tributaries, at approximately right angles, develops a trellis pattern.
A trellis drainage pattern develops where hard and soft rocks exist parallel to each other.
• A rectangular drainage pattern develops on a strongly jointed rocky terrain.
• The radial pattern develops when streams flow in different directions from a central peak
or dome like structure.

Q18. Who originally proposed the concept of payments banks?


A. Bimal Jalan Committee
B. Nachiket Mor Committee
C. P J Nayak Committee
D. YV Reddy Committee

15
©Tathagat GS-Prep Ph: 8431824230/9113994724
ಗೆಲುವು GROUP C TEST SERIES

Ans: B
Recommendations of the Nachiket Mor Committee UPSC
Several recommendations were made by the Nachiket Mor Committee, one of which was to
provide a universal bank account to all Indians above 18 years of age by January 01, 2016.
• Emphasized the importance of Aadhaar as the main force behind the process of increasing
the number of bank accounts.
• Monitoring at the district level, such as deposits and advances as a percentage of GDP.
• By January 1, 2016, all Indians over the age of 18 will have access to universal bank
accounts.
• 50% priority sector lending target modified.

What is Payment Bank?


▪ A payments bank (Airtel Payments Bank, India Post Payments Bank, etc.) is like any other
bank, but operating on a smaller or restricted scale.
▪ Credit risk is not involved with the Payments Bank. It can carry out most banking
operations but cannot advance loans or issue credit cards.
▪ It can accept demand deposits only i.e. savings and current accounts, not time deposits.
▪ The Payment Banks cannot set up subsidiaries to undertake non-banking financial services
activities.
▪ A committee headed by Dr. Nachiket Mor recommended setting up of 'Payments Bank' to
cater to the lower income groups and small businesses.

16
©Tathagat GS-Prep Ph: 8431824230/9113994724
ಗೆಲುವು GROUP C TEST SERIES

▪ Benefits: Expansion of rural banking, access to diversified services, social & financial
inclusion are some of the benefits.
▪ Challenges: Lack of customer awareness, lack of incentives for agents, lack of
infrastructure, technological issues are some of the challenges.

Q19. Match the following well known ecological movements in India to what they are associated
with (persons/places):
Movement: Person/place
1. Navdanya Movement: Vandana Shiva
2. Silent Valley Movement: Kunthipuzha River
3. Appiko Movement: Western Ghats
Select the correct answer using the codes below.
A. 1 and 2 only
B. 2 and 3 only
C. 3 only
D. 1, 2 and 3

Ans: D
1. Vandana Shiva founded Navdanya in 1982, an organisation promoting biodiversity conservation
and organic farming. The organisation has not only helped create markets for farmers, but also
promoted quality food for consumers, connecting the seed to the cooked food, as a part of
Navdanya movement.

2. The Silent Valley hydroelectric project was to dam the Kunthipuzha River, submerging the
entire biosphere reserve and destroying its four-million-year-old rainforests. In 1980, the M.G.K.
Menon Committee set up to review the project, came out with a recommendation to scrap it. With
40 per cent of its so-called surplus power being supplied to other parts and many villages of Kerala
waiting to be electrified, this grassroots movement became the bedrock of Indian environmental
activism

3. The movement happened in the Uttara Kannada district of Karnataka in the Western Ghats.
After independence, the government also began felling trees for profits and the Forest Department,
which continued the colonial forest policy, converted the primeval tropical forests into
monoculture teak and eucalyptus plantations. A group of youth in Balegadde village, objected to
moves to establish teak plantations, wrote to forest officials to stop clearing the natural forest. But
this appeal was overlooked. Then the villagers decided to launch a movement.

Q20. Most of the world’s deserts are located in the western margins of continents in the subtropics.
Which of these reasons explain this phenomenon?

17
©Tathagat GS-Prep Ph: 8431824230/9113994724
ಗೆಲುವು GROUP C TEST SERIES

1. The tropical easterly winds become dry by the time they reach the western margins of the
continents and so they bring no rainfall.
2. The presence of cold ocean currents along the western shores of continents leads to the
development of high pressure over the water surface in these regions preventing rainfall.
Which of the above is/are correct?
A. 1 only
B. 2 only
C. Both 1 and 2
D. None

Ans: C
The prevailing winds in the tropics are tropical easterly winds. The tropical easterly winds become
dry by the time they reach the western margins of the continents and so they bring no rainfall.
Thus, the region becomes devoid of moisture which causes dry conditions leading to formation of
deserts. The presence of cold ocean currents along the western shores of continents leads to the
development of high pressure over the water surface. This high pressure leads to subsidence of air
hinders cloud formation

Q21. A traditional style of Bamboo drip irrigation is practiced in which of the following states of
India?
A. Meghalaya
B. Sikkim
C. Arunachal Pradesh
D. Odisha

Ans: A
For more than 200 years tribal farmers of Megalaya, have been using an indigenous technique of
bamboo drip irrigation to irrigate their plantation crops. These farmers of the Jaintia and Khasi hill
areas have developed this system of tapping springs and stream water to grow betal leaves, black
pepper and arecanut.

The system is so embedded in the indigenous set up of tribal farmers due to the following factors:
Terrain and water availability: The topology of the region is hilly with steep slopes and rock
boulders. The soil depth on these hills is low and has poor water retention capacity. Though the
region gets plenty of rain during the monsoon season, irrigation becomes a necessity during the
dry season. Bamboos as a resource: Meghalaya is richly endowed with bamboo forests. Its
abundance and multiple uses have led bamboo to play a pivotal role in the socio-economic and
cultural life of the tribal people of the state. The 200-year-old system is found in the ‘war’ areas
of Meghalaya but is more prevalent in the ‘war’ Jaintia hills than in the ‘war’ Khasi hills.

18
©Tathagat GS-Prep Ph: 8431824230/9113994724
ಗೆಲುವು GROUP C TEST SERIES

Q22. Which of these disease(s) are transmitted by water or food?


1. Cholera
2. Measles
3. Typhoid
4. Hepatitis B
Select the correct answer using the codes below.
A. 1, 3 and 4 only
B. 2 and 3 only
C. 1 and 4 only
D. 1, 2, 3 and 4

Ans: A

Q23. Consider the following statements.


1. Prarthana Samaj was founded by Aatma Ram Pandurang with an aim to make people
believe in one God and worship only one God.
2. The main reformers of Prarthana Samaj were those who advocated reforms of the social
system of the Hindus.

19
©Tathagat GS-Prep Ph: 8431824230/9113994724
ಗೆಲುವು GROUP C TEST SERIES

Select the correct answer using the codes below.


A. 1 only
B. 2 only
C. Both 1 and 2
D. Neither 1 nor 2

Ans: C
▪ The Prarthana Samaj was established in Bombay by Dr. Atma Ram Pandurang in 1876
with the objective of rational worship and social reform.
▪ The two great members of this Samaj were R.C. Bhandarkar and Justice Mahadev
Govind Ranade.
o They devoted themselves to the work of social reform such as inter-caste dining,
inter-caste marriage, widow remarriage and improvement of the lot of women and
depressed classes.
▪ The four-point social agenda of Prarthana Samaj were
o Disapproval of caste system
o Women education
o Widow remarriage
o Raising the age of marriage for both males and females
▪ Mahavdev Govind Ranade was the founder of the Widow Remarriage Association
(1861) and the Deccan Education Society.
o He established the Poona Sarvajanik Sabha as well.
o To Ranade, religious reform was inseparable from social reform.
o He also believed that if religious ideas were rigid there would be no success in
social, economic and political spheres.
o Although Prarthana Samaj was powerfully influenced by the ideas of Brahmo
Samaj, it did not insist upon a rigid exclusion of idol worship and a definite break
from the caste system.

Q24. Consider the following statements.


1. Kepler gave a theory that explained the reason for the motion of planets.
2. According to Kepler’s law, the cube of the mean distance of a planet from the Sun is
proportional to the square of its orbital period.
3. Kepler’s law says that in a planetary orbit, the line joining the planet and the Sun sweep
equal areas in equal intervals of time.
Which of the above is/are correct?
A. 1 and 2 only
B. 2 and 3 only
C. 1, 2 and 3
D. 3 only

20
©Tathagat GS-Prep Ph: 8431824230/9113994724
ಗೆಲುವು GROUP C TEST SERIES

Ans: B
By the 16th century, a lot of data on the motion of planets had been collected by many astronomers.
Based on these data, Johannes Kepler derived three laws, which govern the motion of planets. It
is important to note that Kepler could not give a theory to explain the motion of planets. It was
Newton who showed that the cause of the planetary motion is the gravitational force that the
Sun exerts on them. These are called Kepler’s laws. These are:
1. The orbit of a planet is an ellipse with the Sun at one of the foci, as shown in the figure
given below. In this figure O is the position of the Sun.
2. The line joining the planet and the Sun sweep equal areas in equal intervals of time. Thus,
if the time of travel from A to B is the same as that from C to D, then the areas OAB and
OCD are equal.
3. The cube of the mean distance of a planet from the Sun is proportional to the square of its
orbital period T. Or, r3/T2 = constant.

Q25. Which of these countries, often seen in news, border the Mediterranean Sea?
1. Syria
2. Lebanon
3. Egypt
4. Jordan
Select the correct answer using the codes below.
A. 1, 2 and 3 only
B. 2, 3 and 4 only
C. 1 and 4 only
D. 1 and 2 only

Ans: A

21
©Tathagat GS-Prep Ph: 8431824230/9113994724
ಗೆಲುವು GROUP C TEST SERIES

Q26. Which of the following fuels has the highest calorific value among the ones listed below:
A. Wood
B. Coal
C. Compressed Natural Gas
D. Hydrogen

Ans: D

Q27. Consider following statements regarding India’s economy.


1. Agricultural exports crossed the $50 bn mark for the first time in 2021-22.
2. India is the top remittance receiver in the world in 2022 followed by Mexico and China.
3. India has been the largest recipient of FDI in the world for the last five years.
Choose incorrect statements.
A. 1 and 2 only
B. 3 only
C. 1 only
D. 1, 2 and 3

Ans: B
As per the DGCI&S data, the country's agricultural products exports had grown by 19.92 percent
in the latest FY of 2022 to touch USD 50.21 billion.
Remittance inflow in India in 2022 is expected to top the $100 billion mark for the first time ever.
It will continue to retain the top spot in the list of countries with the highest remittance receipts, a
report by World Bank said. India will be followed by Mexico with $60 billion in remittance
receipts. At $51 billion, China, which occupied the second spot earlier will receive the third-
highest remittances this year, the report added.
India jumped one position to 7th among the top recipients of foreign direct investment (FDI) in
2021 despite FDI inflows into the country declining, according to the United Nations Conference
on Trade and Development (UNCTAD). Hence, third statement is wrong.

Q28. Consider the following about the mountain passes of Western Ghats.
1. Thal Ghat links Kerala to Tamil Nadu.
2. Bhor Ghat links Mumbai to Pune.
3. Pal Ghat connects Nasik to Mumbai.
Select the correct answer using the codes below.
A. 1 and 2 only
B. 2 only
C. 1 and 3 only
D. 1, 2 and 3

22
©Tathagat GS-Prep Ph: 8431824230/9113994724
ಗೆಲುವು GROUP C TEST SERIES

Ans: B
The Western Ghat Mountain Range or Sahyadri Range is the India's largest mountain range after
the Himalayas which extend from the Tapi river valley to the Nilgiri. It is divided into two parts-
the North Sahyadri and the south Sahyadri.

Some of the important passes of Western Ghats:


• Thal Ghat: It links Nasik to Mumbai.
• Bhor Ghat: It links Mumbai to Rune.
• Pal Ghat: It connects Kerala to Tamil Nadu (link Kochi to Chennai).
• Senkota pass: It is located between the Nagercoil and the Cardmom hills links
Thiruvanathpuram and Madurai

Q29. The country with the longest coastline on the Indian Ocean is
A. Singapore
B. Indonesia
C. Bangladesh
D. India

Ans: D
The coastal length of Myanmar is 2300 km. India is surrounded by Arabian Sea in West, Bay of
Bengal in East and Indian Ocean in south. Including the coastal length of island territories and
mainland it has coastal length of 7561.6km. It is the country with the longest boundary on the
Indian Ocean. The Indian landmass has a central location between the East and the West Asia.
India is a southward extension of the Asian Continent. No other country has a long coastline on
the Indian Ocean as India has and indeed, it is India’s eminent position in the Indian Ocean which
justifies the naming of an Ocean after it.

Q30. Which of the following is/are classified as primary macronutrients for plants?
1. Phosphorous
2. Boron
3. Zinc
4. Nitrogen
Select the correct answer using the codes below.
A. 2 and 3 only
B. 2 and 4 only
C. 1 and 3 only
D. 1, 2 and 3 only

Ans: B
There are 16 elements essential to growth of crop plants:

23
©Tathagat GS-Prep Ph: 8431824230/9113994724
ಗೆಲುವು GROUP C TEST SERIES

• Supplied by air and water: carbon, hydrogen, oxygen


• Macronutrients: nitrogen, phosphorous, potassium
• Secondary Nutrients: calcium, magnesium, sulfur Micronutrients: boron (B), chlorine (Cl),
copper (Cu), iron (Fe), manganese (Mn), molybdenum (Mo), and zinc (Zn).
Micronutrients are essential elements that are used by plants in small quantities. For
most micronutrients, crop uptake is less than one pound per acre. In spite of this low requirement,
critical plant functions are limited if micronutrients are unavailable, resulting in plant
abnormalities, reduced growth and lower yield.

Q31. With respect to the Statutory liquidity ratio (SLR), consider the following statements.
1. SLR is governed by the provisions of the Banking Regulation Act, 1949.
2. A reduction in the limit of SLR will increase the bank credit.
3. State Development Loans do not qualify for SLR.
Choose the correct Statements only.
A. 2 only
B. 1 and 2 only
C. 2 and 3 only
D. 1, 2 and 3 only

Ans: B
State Development Loans qualify for Statutory Liquidity Ratio (SLR) status, that is a proportion
of deposits to be maintained in liquid sovereign securities.

Q32. Which of the following countries in the West Asia border both the Red Sea and the Persian
Gulf?
1. Yemen
2. Saudi Arabia
3. Oman
Which of the above is/are correct?
A. 2 only
B. 3 only
C. 1 and 2 only
D. 1, 2 and 3

Ans: A

Q33. With respect to government securities, choose the correct option -


A. Treasury Bills are issued at a discount & redeemed at face value at maturity. Treasury bills
are zero-coupon securities & pay interest.

24
©Tathagat GS-Prep Ph: 8431824230/9113994724
ಗೆಲುವು GROUP C TEST SERIES

B. Cash Management Bills have the generic character of T-bills but are issued for maturities
of less than 91 days.
C. State Development loans are loans raised by state government from the market. They are
auctioned by SBI.
D. The maturity of Dated Securities and State Development Loans is more than one year so,
these Govt. securities are also traded in the Capital Market like BSE/NSE.

Ans: D
T-bills pay no interest. CMB’s maturity is less than one year. SDLs are also auctioned by RBI
only.

Q34. The Gulf of Aden is sandwiched between which of these nations?


A. Yemen and Somalia
B. Oman and U.A.E.
C. Iraq and Saudi Arabia
D. Egypt and Jordan

Ans: A

Q35. Wangala is a post-harvest festival celebrated in the state of


A. Meghalaya
B. Nagaland
C. Arunachal Pradesh
D. Sikkim

Ans: A
It is a post-harvest festival that honors Misi Saljong. The ritulas are done by the Nokma or the
Principal of the tribe and the offerings consist of cooked rice, brewed beer as well as vegetables.
The music are the highlight and the festival is also called the 100 drum festival.

Zino festival – Arunachal Pradesh - A iconic outdoor music festival in one of India’s most remote
as well as picturesque locations (think rich paddy areas and pine clad mountains), Ziro features a
combination of various indie bands from across the world and also top individual acts from
throughout northeast India. It’s one of the greatest outdoor songs events in India

Q36. Spot the incorrect statement about the difference between the writ jurisdiction of the Supreme
Court and High Courts in India.
1. The Supreme Court can issue writs not only for the enforcement of Fundamental Rights
but also for any other purpose, whereas high courts can issue writs only for the purpose of
enforcement of Fundamental Rights.

25
©Tathagat GS-Prep Ph: 8431824230/9113994724
ಗೆಲುವು GROUP C TEST SERIES

2. High Courts can issue the writ of Injunction, whereas the Supreme Court cannot issue the
writ of Injunction.
3. The Supreme Court can issue writs only in the case of appeal, whereas high courts can
issue writs only when the party directly approaches it.
4. High Courts can issue writs to enforce Fundamental Rights and any other purpose. In
contrast, the Supreme Court can issue writs only to enforce Fundamental Rights.
Choose the correct option
A. 1 and 2.
B. 1,2 and 3.
C. 2 and 3.
D. 4 only

Ans: B
Writs are constitutional remedies and both - High Courts and the Supreme Court enjoy jurisdiction
over this. Articles 226 vests the High Courts with the power to issue writs in case of both - legal
and fundamental rights. However, despite being the apex court, the Supreme Court only issues
writs in cases related to fundamental rights. Hence, the writ jurisdiction of the High Courts is
greater than that of the Supreme Court.

How Writ Jurisdiction of the Supreme Court Differs from that of the High Court?
Where Article 32 of the Indian Constitution empowers the Supreme Court to issue writs; Article
226 empowers the High Courts of India. However, there are a few differences between the writ
jurisdiction of both the courts which are given in the table below:
Difference Supreme Court High Court

Purpose To only enforce To enforce fundamental rights but also for other
fundamental rights purposes (The expression ‘for any other purpose’
refers to the enforcement of an ordinary legal right)

Territorial Against a person or • Against a person residing, government or


Jurisdiction government throughout authority located within its territorial
the territory of India jurisdiction only
Or
• Outside its territorial jurisdiction only if the
cause of action arises within its territorial
jurisdiction

26
©Tathagat GS-Prep Ph: 8431824230/9113994724
ಗೆಲುವು GROUP C TEST SERIES

Power Article 32 is a Discretionary-May refuse to exercise its power to


fundamental right- the issue writs
Supreme Court may not
refuse to exercise its
power to issue the writs

Q37. Mark the correct statements related to the Central Administrative Tribunal are correct?
1. It is a statutory body.
2. Its members are drawn from the administrative background only.
3. It is not bound by the procedure prescribed in the code of civil procedure.
4. Its jurisdiction covers the members of All India Services and Central Services and Central
Government posts.
5. It was set up in 1985.
Choose the correct option
A. 2, 3 and 5.
B. 1 and 4
C. 1,3,4 and 5.
D. 2 and 3

Ans: C
Central Administrative Tribunal
▪ About: It had been established under Article 323 - A of the Constitution.
o It provides for adjudication of disputes and complaints with respect to
recruitment and conditions of service of persons appointed to public services and
posts in connection with the affairs of the Union or other authorities under the
control of the Government.
▪ Legal Framework: In pursuance of Article 323-A, Parliament passed the Administrative
Tribunals Act in 1985.
o The act authorizes the Central government to establish one CAT and the state
administrative tribunals.
o This act opened a new chapter in the sphere of providing speedy and inexpensive
justice to the aggrieved public servants.
o CAT was set up during the Prime Minister ship of Rajeev Gandhi.
▪ Benches: There are 19 Benches in the CAT all over India.
▪ Objective and Composition: The CAT is a specialist body consisting of Administrative
Members and Judicial Members who by virtue of their specialized knowledge are better
equipped to dispense speedy and effective justice.
o A Chairman who has been a sitting or retired Judge of a High Court heads the
CAT.
27
©Tathagat GS-Prep Ph: 8431824230/9113994724
ಗೆಲುವು GROUP C TEST SERIES

▪ Operating Principles: The Tribunal is guided by the principles of natural justice in


deciding cases and is not bound by the procedure prescribed by the Civil Procedure
Code.
o Under Section 17 of the Administrative Tribunal Act, 1985, the Tribunal has
been conferred with the power to exercise the same jurisdiction and authority in
respect of contempt of itself as a High Court.
▪ Independence: The conditions of service of the Chairman and Members are the same as
applicable to a Judge of High Court as per the Administrative Tribunals (Amendment)
Act, 2006.
▪ Appeals against Orders: The orders of the CAT are challenged by way of a Writ
Petition under Article 226/227 of the Constitution before the respective High Court in
whose territorial jurisdiction the Bench of the Tribunal is situated.

Q38. Which one of the following devices calls the minister’s attention to a matter of public
importance?
A. Half an hour discussion
B. Calling attention notice
C. Short duration discussion
D. Adjournment Motion

Ans: B
Calling Attention Motion: Introduced in the Parliament by a member, Calling Attention Motion
is a process to call the attention of a minister with respect to a matter of urgent public importance
and to seek an authoritative statement from him on that matter.

Calling Attention Motion was introduced in India in 1954. It can be introduced by any house in
the Parliament, whether it is Lok Sabha or a Rajya Sabha.

Q39. The Constitution says that the State Council of Ministers hold office during the pleasure of
the Governor. The words “during the pleasure of the Governor” in reality mean
A. Pleasure of the President
B. Pleasure of the Prime Minister
C. Pleasure of the Chief Minister
D. Pleasure of the Legislative Assembly

Ans: C
The doctrine of pleasure originated in England as a special prerogative (or right/ privilege) of the
British crown. Under the law a servant of the crown holds office as per the crown and can be
dismissed at the crown's will or pleasure. The doctrine of pleasure in India is embodied in Article

28
©Tathagat GS-Prep Ph: 8431824230/9113994724
ಗೆಲುವು GROUP C TEST SERIES

310 (1) of the constitution. The doctrine is accepted in India partially as compared to the one
accepted under the crown and is subjected to the provisions given in Article 311.

ARTICLE 310 (1) states: except the provisions provided (or expressly excluded) by the
Constitution, every person who is a member of a defense service or of a civil service/ of an all
India service/ holds any post connected with defense/holds any civil post under the Union, holds
office during the pleasure of the President. Also, every person who is a member of a civil service
of a State or holds any civil post under a State holds office during the pleasure of the Governor of
the State.
A. Pleasure of the president – in India the Attorney General, Civil Services Personal, Prime
Minister and Council of Ministers serve at the pleasure of the President.
B. Pleasure of Prime minister - the PM determines the general direction of the nation’s
government related activities & holds office at pleasure of president.
C. Pleasure of the chief minister – the CM holds office under the pleasure of the governor and
can advise him on different matters.
D. Pleasure of legislative assembly - The Constitution vests in the governor the executive
powers of the state government, i.e. the Council of Ministers will remain in power during
the 'pleasure' of the governor, but in the real sense it means the pleasure of obtaining a
majority in the Legislative Assembly.

Q40. Under the constitution of India, which one of the following is not a fundamental duty?
A. To vote in public elections
B. To develop the scientific temper
C. To safeguard public property
D. To abide by the Constitution and respect its ideals

Ans: A
The list of 11 Fundamental Duties under Article 51-A to be obeyed by every Indian citizen is given
in the table below:
S.No 11 Fundamental Duties

1. Abide by the Indian Constitution and respect its ideals and institutions, the National Flag
and the National Anthem

2. Cherish and follow the noble ideals that inspired the national struggle for freedom

3. Uphold and protect the sovereignty, unity and integrity of India

4. Defend the country and render national service when called upon to do so

29
©Tathagat GS-Prep Ph: 8431824230/9113994724
ಗೆಲುವು GROUP C TEST SERIES

5. Promote harmony and the spirit of common brotherhood amongst all the people of India
transcending religious, linguistic and regional or sectional diversities and to renounce
practices derogatory to the dignity of women

6. Value and preserve the rich heritage of the country’s composite culture

7. Protect and improve the natural environment including forests, lakes, rivers and wildlife
and to have compassion for living creatures

8. Develop scientific temper, humanism and the spirit of inquiry and reform

9. Safeguard public property and to abjure violence

10. Strive towards excellence in all spheres of individual and collective activity so that the
nation constantly rises to higher levels of endeavour and achievement

11. Provide opportunities for education to his child or ward between the age of six and
fourteen years. (This duty was added by the 86th Constitutional Amendment Act,
2002)

Q41. Which of the following type of forest occupies the largest area in India?
A. Tropical Moist Deciduous Forest
B. Sub-tropical Dry Evergreen Forest
C. Montane Wet Temperate Forest
D. Tropical Wet Evergreen Forest

Ans: A
Tropical moist deciduous is followed by tropical dry deciduous. So, deciduous forests cover the
largest forest area in India, more than 60%

30
©Tathagat GS-Prep Ph: 8431824230/9113994724
ಗೆಲುವು GROUP C TEST SERIES

Q42. Which of the following is not an area of coral reef?


A. Gulf of Mannar
B. Gulf of Cambay
C. Lakshadweep
D. Andaman and Nicobar Islands

Ans: B
CORAL REEF
How are they formed?
Coral polyps are tiny and fleshy sea anemones that live in tropical and subtropical oceans and seas.
They live in shallow waters along with microscopic algae called Zooxanthellae, with which they
share a symbiotic relationship. These algae have photosynthesis abilities that feed the coral polyps
with carbon compounds which give them energy. In return, the polyps provide protection to
Zooxanthellae.

Where are coral reefs found in India?


India is recorded to have around 7,517 km of coastline but only 6,100 km represents the mainland
coastline. The three major types of coral reefs found in India are fringing, barrier and atolls.
Coral reefs in India are found in a lot of areas including the Gulf of Kutch, Gulf of Mannar, Palk
Bay, Andaman & Nicobar and Lakshadweep Islands. The Gulf of Kutch in the northwest has some
of the most northerly reefs in the world.

31
©Tathagat GS-Prep Ph: 8431824230/9113994724
ಗೆಲುವು GROUP C TEST SERIES

Patches of coral reefs are also found in Ratnagiri, Malvan and Redi, south Bombay and at the
Gaveshani Bank located in the west of Mangalore.

Types of coral reefs in India


India has three major prevailing types of coral reefs:
1. Fringing reefs
Fringing reefs evolve and develop near the continent and remain close to the coastline. These reefs
are separated from the coastline by small, shallow lagoons. They are the most commonly found
reefs in the world.
2. Barrier reefs
Barrier reefs are found offshore on the continental shelf. They usually run parallel to the coastline
at some distance. A deep and wide lagoon is located between the coastline and the barrier reef.
3. Atolls
Atolls are formed on mid-oceanic ridges. They are shaped circularly or elliptically and are
surrounded by seas on all four sides and have shallow waters in the center called a lagoon.

Q43. Biological indicator of pollution of sulfur dioxide is: -


A. moss
B. smoke
C. Braophyta
D. None of these

Ans: A
Due to the sensitivity of Lichens to sulphur dioxide, it is very useful as a biological indicator of
Sulphur-dioxide pollution. The efficient absorption systems of Lichens result in rapid
accumulation of sulphur during high levels of sulphur dioxide pollution.

Why in News?
Recently, the Ministry of Power has informed Lok Sabha that, for compliance with Sulphur
dioxide (SO2) emission norms, Thermal Power Plants are installing Flue Gas Desulphurisation
(FGD) equipment.The Ministry in September 2022 had extended the deadline for Coal-Fired
power plants to install FGD to cut sulphur emissions by two years.

What is the Categorisation of Power Plants for Installing FGD?


Category Location/Area Timelines for compliance

32
©Tathagat GS-Prep Ph: 8431824230/9113994724
ಗೆಲುವು GROUP C TEST SERIES

Category A Within 10 km radius of Upto 31st December 2024


National Capital Region
(NCR) or cities having million
plus population (as per 2011
census of India)
Category B Within 10 km radius of Upto 31st December 2025
Critically Polluted Areas or
Non-attainment cities (as
defined by CPCB)
Category C Other than those included in Upto 31st December 2026
category A and B

What is Flue Gas Desulfurization (FGD)?


▪ About:
o FGD is the process of removing sulfur compounds from the exhaust emissions
of fossil-fueled power stations.
o This is done through the addition of absorbents, which can remove up to 95% of
the sulphur dioxide from the flue gas.
o Flue gas is the material emitted when fossil fuels such as coal, oil, natural gas, or
wood are burned for heat or power.

Q44. An aquatic plant was introduced to the US to remove pollution in Indian aquatic bodies as a
troublesome weed. what is its name?
A. hawthorn
B. Egiolps
C. Hyacinth or Eichhornia
D. Pistia

Ans: C
Water-Hyacinth (Eichhornia crassipes) is a plant that is native to the Amazon basin in the
Americas. It was introduced around the world and in India since it was a very pretty plant.
However, it grew very fast and forming dense mats that can spread across water surfaces
eventually choking the entire water body. It destroyed native wetlands and waterways, killing
native fish and other wildlife.

Q45. Which award is given to those who show indomitable courage in the field of wildlife
conservation?
A. Indira Gandhi Environment Award
B. Medini Award Scheme
C. Amrita Devi Bishnoi Award

33
©Tathagat GS-Prep Ph: 8431824230/9113994724
ಗೆಲುವು GROUP C TEST SERIES

D. Pitambar Pant National Award

Ans: C
The Government of India instituted an 'Amrita Devi Bishnoi National Award for Wildlife
Conservation' in the memory of Amrita Devi Bishnoi, who in 1731 sacrificed her life along with
363 others for the protection of 'khejri' trees in Khejarli village near Jodhpur in Rajasthan. Its main
objective is to preserve wildlife and protection of trees.

Q46. Which of the following protocols is not correctly related to its issue?
A. 1987 Montreal Protocol - Ozone Depleting Substances
B. 1979 Bone Conference - Conservation of Migratory Species
C. 1989 Basel Convention - Regulation of transboundary movement, transit, handling and use
of living modified organisms
D. 1998 Rotterdam Conference - Consent in the international trade of some hazardous
chemicals and pesticides

Ans: C
The Basel Convention directs a “Prior Consent Approval” method to control the transboundary
movements of harmful and other wastes to reduce their adverse effects on the environment. Basel
Convention states that the transfer or movement of harmful and other wastes among the -party
nations is illegal.

Living Modified Organisms (LMOs) including GM crops are regulated which are governed
under the Cartagena Protocol on Biosafety (CPB). Transboundary movement of LMOs/ GM
crops is also regulated by the Protocol. The Protocol was adopted by the CBD in September 2000
and came into force in September 2003.

Q47. Whose register is the montrix record?


A. Dangerous foreign species
B. endangered wild species
C. Aquatic places at risk under anthropogenic activities
D. None of these

Ans: C
The Montreux record is a register of wetland sides on the list of wetlands of international
importance where changes in ecological character have occurred, are occurring, or are likely to
occur as a result of technological developments, pollution or other human interference. Indian sites
in the like Keoladeo National Park-Rajasthan, Loktak Lake – Manipur are included in Montreux
Record.

34
©Tathagat GS-Prep Ph: 8431824230/9113994724
ಗೆಲುವು GROUP C TEST SERIES

Ramsar Sites in India are declared under the Ramsar Convention, established by UNESCO in
1971. The Ramsar Convention was signed with the aim of making wise use of the wetlands with
national, international, and also local corporations for sustainable development of the overall
world.

Q48. Thin wires can be drawn from a piece of metal. Which property of the metal is it responsible
for?
A. ductility
B. hardness
C. malleability
D. conductivity

Ans: A
The property of metal by which it can be drawn into wires is called ductility.

Q49. In the presence of sunlight, plants prepare their food (glucose) with CO2 and water. What is
this reaction?
A. Displacement
B. Disintegration
C. Photochemical
D. precipitation

Ans: C
Photochemical is the process by which plants use sunlight, water, and carbon dioxide to create
oxygen and energy in the form of sugar.

Q50. Which of the following was not related to the principles of atomic structure?
A. John Dalton
B. Ernest Rutherford
C. Niels Henry David Bohr
D. J Chadwick

Ans: D
The Correct Answer is J Chadwick. The atomic structure consists of protons, electrons, and
neutrons. The nucleus of an atom consists of Protons and Neutrons. J Chadwick was not associated
with the atomic model while the remaining three associated with the atomic model.

35
©Tathagat GS-Prep Ph: 8431824230/9113994724
ಗೆಲುವು GROUP C TEST SERIES

Q51. The first site of the ancient Indus Valley Civilisation in India to get the UNESCO World
Heritage Site tag is
A. Lothal
B. Dholavira
C. Kalibangan
D. Mohenjodaro

Ans: B
Dholavira, the archaeological site of a Harappan-era city, received the UNESCO world heritage
site tag. While Dholavira became the fourth site from Gujarat and 40th from India to make the list,
it is the first site of the ancient Indus Valley Civilisation (IVC) in India to get the tag. After
Mohenjo-Daro, Ganweriwala and Harappa in Pakistan and Rakhigarhi in Haryana of India,
Dholavira is the fifth largest metropolis of IVC. A World Heritage Site is a landmark or area,
selected by the UN Educational, Scientific and Cultural Organization (UNESCO) for having
cultural, historical, scientific or other forms of significance, which is legally protected by
international treaties.

Q52. Consider the following about certain natural preservatives used in households.
1. Sugar reduces the moisture content of food which inhibits the growth of bacteria which
spoil food.
2. Use of oil and vinegar prevents spoilage of pickles because bacteria cannot live in such an
environment.
Select the correct answer using the codes below.
A. 1 only
B. 2 only
C. Both 1 and 2
D. Neither 1 nor 2

Ans: C
Common salt has been used to preserve meat and fish for ages. Meat and fish are covered with dry
salt to check the growth of bacteria. Salting is also used to preserve amla, raw mangoes, tamarind,
etc.

Preservation by Sugar - Jams, jellies and squashes are preserved by sugar. Sugar reduces the
moisture content which inhibits the growth of bacteria which spoil food.
Preservation by Oil and Vinegar - Use of oil and vinegar prevents spoilage of pickles because
bacteria cannot live in such an environment. Vegetables, fruits, fish and meat are often preserved
by this method.

Q53. Thermosetting plastics are generally stronger than thermoplastic materials because they have

36
©Tathagat GS-Prep Ph: 8431824230/9113994724
ಗೆಲುವು GROUP C TEST SERIES

1. three-dimensional network of cross-linked bonds


2. better suitability to high-temperature applications as their covalent bonds between polymer
chains cannot be broken easily
Which of the above is/are correct?
A. 1 only
B. 2 only
C. Both 1 and 2
D. None

Ans: C
Thermosetting plastics are generally stronger than thermoplastic materials due to the three-
dimensional network of bonds (crosslinking), and are also better suited to hightemperature
applications up to the decomposition temperature since they keep their shape as strong covalent
bonds between polymer chains cannot be broken easily. Conventional thermoset plastics or
elastomers cannot be melted and re-shaped after they are cured. This usually prevents recycling
for the same purpose, except as filler material

The higher the crosslink density and aromatic content of a thermoset polymer, the higher the
resistance to heat degradation and chemical attack. Mechanical strength and hardness also improve
with crosslink density, although at the expense of brittleness. They normally decompose before
melting. Hard, plastic thermosets may undergo permanent or plastic deformation under load.
Elastomers, which are soft and springy or rubbery and can be deformed and revert to their original
shape on loading release.

Q54. Which of the following reports were published by the International Monetary Fund (IMF)?
1. World Economic Outlook.
2. Global Financial Stability Report.
3. Global Social Mobility Report.
4. Global Risk Report.
Select the correct answer using the code given below:
1. 1, 2 and 4 only
2. 3 and 4 only
3. 1 and 2 only
4. 1, 2, 3 and 4

Ans: C
Global Social Mobility Report and Global Risk Report are Published by World Economic Forum.

Q55. Which is the best definition of the term “Merchant Discount Rate”?

37
©Tathagat GS-Prep Ph: 8431824230/9113994724
ಗೆಲುವು GROUP C TEST SERIES

A. A special rate under digital India given by the Government to merchants when payments
are made through Point of Sale machines and debit cards.
B. The charge given to a merchant by a bank for accepting payments from his customers
through the bank’s debit cards.
C. The incentive paid by banks to their customers directly when debit cards are used for
financial transactions.
D. The charge given by a bank to the merchant for ensuring payments through the debit cards.

Ans: B
Merchant Discount Rate is a fee charged by a bank from a merchant for accepting payments from
customers through credit and debit cards. Simply, it is the cost paid by a merchant to a bank for
accepting payment and is expressed in percentage of the transaction amount. It is applicable for
online transactions and QR-based transactions. The amount paid by the merchant is distributed
among the bank enabling transaction, vendor that installs the point of sale machine and the network
provider such as MasterCard, Visa, RuPay etc.

Q56. Consider the following statements with reference to the Toll-Operate-Transfer (TOT) model:
1. The right of collection of tolls on existing highway stretches is auctioned.
2. It is applicable only to highways built through public funding.
Which of the statements given above is/are correct?
A. 1 only
B. 2 only
C. Both 1 and 2
D. Neither 1 nor 2

Ans: C
The National Highways Authority of India (NHAI), for monetizing public-funded national
highways proposes to use the Toll-operate-Transfer (ToT) model.
Under this model, the right of collection of user-fee or toll on selected national highway stretches
that have been built through public funding is proposed to be assigned for a 30-year period to
developers and investors against an upfront payment of a lump-sum amount to the government.
During the tenure of the contract, the operation and maintenance would be the responsibility of the
developer.

Q57. Consider the following statements with reference to the World Trade Organization (WTO):
1. The General Council is the highest decision-making body.
2. All major decisions are made by consensus.
3. One of its objectives is to reduce the dis-equilibrium in the international balance of
payments of its members.
Which of the statement(s) given above is/are correct?

38
©Tathagat GS-Prep Ph: 8431824230/9113994724
ಗೆಲುವು GROUP C TEST SERIES

A. 2 only
B. 1 and 3 only
C. 3 only
D. 2 and 3 only

Ans: A
The World Trade Organization (WTO) is an intergovernmental organization that regulates and
facilitates international trade. With effective cooperation in the United Nations System,
governments use the organization to establish, revise, and enforce the rules that govern
international trade.
Organizational structure
The highest authority of the WTO is the Ministerial Conference, which must meet at least every
two years. The Ministerial Conference met most recently in June 2022 in Geneva. Hence,
statement 1 is not correct.
In between each Ministerial Conference, the daily work is handled by three bodies whose
membership is the same; they only differ by the terms of reference under which each body is
constituted.
• The General Council
• The Dispute Settlement Body
• The Trade Policy Review Body
All major decisions are made by the membership as a whole. Hence, statement 2 is correct.
IMF aims to reduce the dis-equilibrium in the international balance of payments of its members.
Hence, statement 3 is not correct.

Q58. Consider the following statements.


1. M.K. Gandhi declared Vinoba Bhave as his spiritual heir.
2. Vinoba Bhave supported Gandhi’s concept of gram swarajya and was the first individual
Satyagrahi.
Which of the above is/are correct?
A. 1 only
B. 2 only
C. Both 1 and 2
D. Neither 1 nor 2

Ans: C
Mahatma Gandhi declared Vinoba Bhave as his spiritual heir. He was one of the votaries of
Gandhi’s concept of gram swarajya. After Gandhiji’s martyrdom, Vinobha Bhave undertook
padyatra to spread Gandhiji’s message covered almost the entire country. In 1940 Gandhi chose
Vinoba to be the first Satyagrahi i.e. non-violent resister, to offer nonviolent resistance to the
British

39
©Tathagat GS-Prep Ph: 8431824230/9113994724
ಗೆಲುವು GROUP C TEST SERIES

Q59. Consider the following about the Tyndall effect.


1. It can indicate the size and density of particles in aerosols and other colloidal matter.
2. Under the effect, the shorter wavelengths of light are transmitted more than the longer
wavelengths.
Which of the above is/are correct?
A. 1 only
B. 2 only
C. Both 1 and 2
D. None

Ans: A
The small sized colloidal particles in air can easily scatter a beam of visible light. This scattering
of a beam of light is called the Tyndall effect. Under the Tyndall effect, the longer wavelengths
are more transmitted while the shorter wavelengths are more diffusely reflected via scattering. The
Tyndall effect is seen when lightscattering particulate matter is dispersed in an otherwise light-
transmitting medium, when the diameter of an individual particle is the range of roughly between
40 and 900 nm, i.e. somewhat below or near the wavelengths of visible light (400–750 nm). In the
forest, mist contains tiny droplets of water, which act as particles of colloid dispersed in air. Dust
and smoke present in the air scatter the beam of light

Q60. Match the following plant diseases with the micro-organisms responsible for them:
Disease: Micro-organism
1. Citrus canker: Fungi
2. Rust of Wheat: Bacteria
3. Yellow Vein Mosaic: Virus
Select the correct answer using the codes below.
A. 3 only
B. 2 and 3 only
C. 1 and 2 only
D. 1, 2 and 3

Ans: A

40
©Tathagat GS-Prep Ph: 8431824230/9113994724
ಗೆಲುವು GROUP C TEST SERIES

Q61. Which of these regions in India is/are well known for the production of copper?
1. Balaghat mines in Madhya Pradesh
2. Khetri mines in Rajasthan
3. Singbhum district of Jharkhand
Which of the above is/are correct?
A. 1 only
B. 2 and 3 only
C. 1 and 2 only
D. 1, 2 and 3

Ans: D
Copper production in India is only about 2 percent of world copper production in view of its
potential reserve limited to 60,000 km2 (2% of world reserve) of which 20,000 km2 area has been
subject to exploration, as of 2012. The Balaghat mines in Madhya Pradesh produce most of India’s
copper. The Singbhum district of Jharkhand is also a leading producer of copper. The Khetri mines
in Rajasthan are also famous for Copper. In India, copper ore has been identified in varying
structural formations as "host rocks" in several geological formations of different geological time
scale. The resource has been identified and explored to varying degree in 14 states of the country
such as Andhra Pradesh, Gujarat, Haryana, Jharkhand, Karnataka, Madhya Pradesh, Maharashtra,
Meghalaya, Orissa, Rajasthan, Sikkim, Tamil Nadu, Uttarakhand and West Bengal.

Q62. Consider the following matches of tiger reserves with the state they are located in.
Tiger Reserve: State
1. Nagarjunsagar-Srisailam Tiger Reserve: Andhra Pradesh

41
©Tathagat GS-Prep Ph: 8431824230/9113994724
ಗೆಲುವು GROUP C TEST SERIES

2. Manas Tiger Reserve: Assam


3. Bandhavgarh National Park: Madhya Pradesh
4. Sariska Wildlife Sanctuary: Rajasthan
Select the correct answer using the codes below.
A. 1 and 2 only
B. 3 and 4 only
C. 2 and 3 only
D. 1, 2, 3 and 4

Ans: D

Q63. What is the correct north-south order of the following major dams in India?
1. Rana-Pratap Sagar
2. Sardar Sarovar
3. Pravara
4. Koyana
Select the correct answer using the codes below.
A. 1 2 3 4
B. 4 3 2 1
C. 2 3 1 4
D. 1 3 2 4

Ans: A
The Rana Pratap Sagar Dam is a gravity masonry dam of 53.8 metres (177 ft) height built on the
Chambal River at Rawatbhata in Rajasthan in India.

The Sardar Sarovar Dam is a concrete gravity dam built on the Narmada River in Navagam near
the town of Kevadiya, Narmada District, in the state of Gujarat

Pravara Dam is a very quiet Dam situated at Ahmednagar district in Maharashtra

The Koyna Dam is the largest dam in Maharashtra. The Dam is located in Koyana Nagar, Satara
District. It is nestled in the Western Ghats on the state highway between Chiplun and Karad.

Q64. Public Union for Civil Liberties v Union of India (1996) case concerned the matter of
A. Breach of privacy
B. Protection of fundamental rights
C. Right against exploitation of marginalized groups
D. Directive principles of state policy

42
©Tathagat GS-Prep Ph: 8431824230/9113994724
ಗೆಲುವು GROUP C TEST SERIES

Ans: A
Public Union for Civil Liberties v Union of India (1996): A public interest litigation was filed in
the wake of the report on “Tapping of politicians phones” by the CBI. So, the Supreme Court
pointed out lack of procedural safeguards in the provisions of the Telegraph Act and laid down
certain guidelines for interceptions.

These guidelines formed the basis of introducing:


• Rule 419A in the Telegraph Rules in 2007.
• The rules prescribed under the IT Act in 2009.

Q65. Consider the following statements.


1. India hosts a majority of global tiger population.
2. In India, Madhya Pradesh has the highest number of tigers, followed by Karnataka and
Uttarakhand.
Select the correct answer using the codes below.
A. 1 only
B. 2 only
C. Both 1 and 2
D. Neither 1 nor 2

Ans: C
On the occasion of Global Tiger Day (July 29), a detailed report on the All-India Tiger Estimation
2022 was released.

Highlights of the report:


• According to the Wildlife Institute of India’s (WII) 5th quadrennial tiger census, India’s
tiger population increased to 3,682 in 2022.
• The numbers in the report, revised from 3167 earlier, reflect a commendable annual
growth rate of 6.1% per annum.
• The largest tiger population of 785 is in MP, followed by Karnataka (563) and
Uttarakhand (560), and Maharashtra (444).
• The tiger abundance within the Tiger Reserve is highest in Corbett (260), followed by
Bandipur (150), Nagarhole (141), Bandhavgarh (135), Dudhwa (135),
• Central India and the Shivalik Hills and Gangetic Plains witnessed a notable increase
in tiger population, while the Western Ghats experienced localised declines,
necessitating targeted monitoring and conservation efforts.
• Some states, including Mizoram, Nagaland, Jharkhand, Goa, Chhattisgarh and
Arunachal Pradesh, have reported disturbing trends with small tiger populations.

43
©Tathagat GS-Prep Ph: 8431824230/9113994724
ಗೆಲುವು GROUP C TEST SERIES

Q66. Consider the following statements.


1. Fungi do not grow on carbon-rich substrates such as glucose.
2. Aspergillus species are common fungi and are found in almost all oxygen-deprived
environments.
Select the correct answer using the codes below.
A. 1 only
B. 2 only
C. Both 1 and 2
D. Neither 1 nor 2

Ans: D
Aspergillus is a fungus whose spores are present in the air we breathe, but does not normally cause
illness.
• Aspergillus species are highly aerobic and are found in almost all oxygen-rich
environments, where they commonly grow as molds on the surface of a substrate, as a
result of the high oxygen tension. Commonly, fungi grow on carbon-rich substrates like
monosaccharides (such as glucose) and polysaccharides (such as amylose).
• In those people with a weakened immune system, damaged lungs or with allergies,
Aspergillus can cause disease. Common Aspergillus infections include invasive
aspergillosis, ABPA CPA and aspergilloma.

Sources of high numbers of Aspergillus spores include air conditioning units, composting and
damp or flood damaged housing & hospital building projects. They do not occur in such dry and
hot conditions, which aren’t ideal for fungi to grow. Members of the genus possess the ability to
grow where a high osmotic pressure exists (high concentration of sugar, salt, etc.).

Q67. If x is a positive integer such that 2x +12 is perfectly divisible by x, then the number of
possible values of x is
A. 2
B. 5
C. 6
D. 12

Ans: C

If 2x+12 is perfectly divisible by x, then 12 must be divisible by x.

Hence, there are six possible values of x: (1,2,3,4,6,12)

44
©Tathagat GS-Prep Ph: 8431824230/9113994724
ಗೆಲುವು GROUP C TEST SERIES

If x = 1 then 2*1+12 = 14 is divisible by 1

If x = 2 then 2*2+12 = 18 is divisible by 2

If x = 3 then 2*3+12 = 18 is divisible by 3

If x = 4 then 2*4+12 = 20 is divisible by 4

If x = 6 then 2*6+12 = 24 is divisible by 6

If x = 12 then 2*12+12 = 36 is divisible by 12

Q68. An outgoing batch of students wants to construct an auditorium worth Rs 42,00,000 for their
college. If the teachers offer to pay 50% more than the student’s contribution and an external
benefactor give three times the teachers contribution, how much should the teachers donate?
A. 9,00,000 Rs
B. 7,35,000 Rs
C. 8,40,000 Rs
D. 6,50,000 Rs

Ans: A

Let’s assume the students contributed X Rs.

So, the teachers contributed 3X/2 Rs

And external benefactor contributed 9X/2 Rs

Summation of all contribution = 42,00,000 = 7X

Or, X = 6,00,000 Rs

Hence, teachers contributed 9,00,000 Rs

Q69. A bought 5 cars, 7 tractors and 4 buses. B bought 6 cars, 8 buses and 14 tractors for an amount
which was half more than what A had paid. What per cent of the total amount paid by A was paid
for the cars?
A. 37.5%
B. 62.5%
C. 50%

45
©Tathagat GS-Prep Ph: 8431824230/9113994724
ಗೆಲುವು GROUP C TEST SERIES

D. None of these

Ans: B

Let the cost of car, tractor and bus be x, y, and z respectively.

5x+7y+4z = Aa ………. (1)

6x+8z+14y = 3A/2

4x + 16/3 z + 28/3 y = Aa ………. (2)

Comparing two equations

5x+7y+4z = 4x+16/3 z + 28/3 y

x = 7/3 y + 4/3 z

3x = 7y+4z

Now required percentage = [5x/(5x+7y+4z)] *100 = 5x/(5x+3x) = 62.5%

Q70. A lady lost her purse. She went 90 m east to a shop before turning to her right. She went 20
m ahead again before turning to her right. Thereafter, she walked 30 m further to reach his uncle’s
place. From there, she went 100 m north to a street where she found her purse. How far from the
starting point did she find her purse?
A. 60m
B. 100m
C. 140m
D. 180m

Ans: B

Q71. Match the following correctly:


1. Parasitic Nutrition a) Mushroom
2. Saprophytic Nutrition b) Cassytha
3. Symbiotic Nutrition c) Fungi and trees
Select the correct answer using the codes given below
A. 1-c, 2-b, 3-a
B. 1-a, 2-c, 3-b

46
©Tathagat GS-Prep Ph: 8431824230/9113994724
ಗೆಲುವು GROUP C TEST SERIES

C. 1-b, 2-a, 3-c


D. 1-b, 2-c 3-a

Ans: C
Parasitic Nutrition: Some heterotrophic plants depend on other plants and animals for nutrition.
Such plants are known as parasitic plants. However, the host has not benefited from the parasite.
For eg., Cuscuta, Cassytha
Saprophytic Nutrition: The saprophytic plants derive nutrition from dead and decaying plants and
animals. They dissolve the dead and decaying matter by secreting digestive juices and absorb the
nutrients. For eg., mushrooms, molds.
Symbiotic Nutrition: When two different plants belonging to two different categories show a close
association, they are termed as symbiotic. In this, both the plants are benefitted from each other.
For eg., the association of fungi and trees.

Q72. Manoj is the brother of Deepak. Rekha is the sister of Seema. Deepak is the son of Rekha.
How is Manoj related to Rekha?
A. Nephew
B. Son
C. Brother
D. Sister

Ans: B

Q73. Which among the following activities are not permitted to payments banks?
A. Remittance services
B. Issuing ATM cards
C. Issuing credit cards
D. Accepting demand deposits

Ans: C

Q74. Each of the six different faces of a cube has been coated with a different colour i.e. V I B G
Y and O. Following information is given:
1. Colours Y O and B are on adjacent faces.
2. Colours I G and Y are on adjacent faces.
3. Colours B G and Y are on adjacent faces.
4. Colours O V and B are on adjacent faces.
Which is the colour of the face opposite to the face coloured with “O”?
A. B
B. V

47
©Tathagat GS-Prep Ph: 8431824230/9113994724
ಗೆಲುವು GROUP C TEST SERIES

C. G
D. I

Ans: C

Q75. Consider the following statements regarding Starlink:


1. It is a product of Space-X which provides satellite-based internet connectivity.
2. It has potential to provide faster speed than a 5G network.
3. For domestic use, it is cheaper compared to optical fibre connectivity.
Which of the following statements is/are correct
A. 1 only
B. 1 and 3 only
C. 1 and 2 only
D. 2 and 3 only

Ans: C
Both statements 1 and 2 are correct. Starlink is a satellite-based internet service provided by
SpaceX, and it has the potential to offer faster internet speeds than 5G networks.
However, statement 3 is incorrect, as Starlink is currently more expensive than traditional fiber-
optic internet connectivity for domestic use.

Q76. Look at this series: 36, 34, 30, 28, 24, ... what number should come next?
A. 20
B. 22
C. 23
D. 26

Ans: D

Q77. Consider the following statements regarding Gaganyaan programme.


1. After launching Gaganyaan Space Mission, India will join USA, Russia and China as the
countries who have launched a human spaceflight mission.
2. It involves sending a two membered crew for 5 to 7 days.
3. It involves demonstration of human spaceflight in the Low Earth Orbit.
Which of the options below is correct?
A. 3 only
B. 1 and 2 only
C. 1 and 3 only
D. 2 and 3 only

48
©Tathagat GS-Prep Ph: 8431824230/9113994724
ಗೆಲುವು GROUP C TEST SERIES

Ans: C
Statement 1 is correct. "Gaganyaan" is India's first human space mission to be launched in 2023.
This will make India the fourth nation to launch a human spaceflight mission after the USA, Russia
and China.
Statement 2 is correct. The mission aims to send a three membered crew for 5 to 7 days. It involves
three flights of which two are unmanned and one is manned.
Statement 3 is correct. It strives to demonstrate human spaceflight in the Low Earth Orbit for short-
term and a sustained Indian human space exploration in the long-run. Indian Space Research
Organization conducted a test of the cryogenic engine for 720 seconds at the ISRO Propulsion
Complex in Tamil Nadu's Mahendragiri.

Q78. January 3, 2007 was Wednesday. What day of the week fell on January 3, 2008?
A. Tuesday
B. Friday
C. Thursday
D. Saturday

Ans: C

Q79. A person travelled a distance of 50 km in 8h. He covered a part of the distance on foot @ of
4km/h and a part on bicycle @ 10km/h. How much distance did he travel on foot?
A. 10
B. 20
C. 30
D. 15

Ans: B

Q80. What was the name of the operation which was recently launched by Central Bureau of
Investigation (CBI) against illicit drug trafficking network?
A. Operation Garuda
B. Operation Himmat
C. Operation Prahar
D. Operation CLEAN

Ans: A
The Central Bureau of Investigation has launched a multi-phase ‘Operation Garuda’ against illicit
drug trafficking network, registering 127 new cases, arresting 175 people and seizing huge
quantities of narcotic drugs.
About ‘Operation Garuda’

49
©Tathagat GS-Prep Ph: 8431824230/9113994724
ಗೆಲುವು GROUP C TEST SERIES

• It was initiated in close coordination with Interpol and Narcotics Control Bureau, for
combating the smuggling of illicit drugs and psychotropic substances, with special focus
on the Indian Ocean region.
o Drug trafficking networks with international linkages require law enforcement
cooperation across international jurisdiction.
• Objectives: It seeks to disrupt, degrade and dismantle drug networks with international
linkages through rapid exchange of criminal intelligence on drug trafficking and
coordinated law enforcement actions across international jurisdiction through Interpol.

Q81. If the difference between compound and simple interest on a certain sum of money for 3
years at 10% per annum is ₹310, what is the sum?
A. ₹8000
B. ₹1000
C. ₹10000
D. ₹5000

Ans: C

Q82. A and B started a business with Rs. 1500 and Rs. 2500 and got a profit of Rs. 800. Half of
the profit is shared equally the remaining is shared according to their investment. Find their profits
respectively.
A. Rs. 400 and Rs. 400
B. Rs. 500 and Rs. 300
C. Rs. 300 and Rs. 500
D. Rs. 350 and Rs. 450

Ans: D

Q83. Tickets numbered 1 to 20 are mixed up and then a ticket is drawn at random. What is the
probability that the ticket drawn has a number which is a multiple of 3 or 5?
A. 1/2
B. 2/5
C. 8/15
D. 9/20

Ans: D

Q84. Supriya can swim in still water at a speed of 10 kmph. If speed of the current would have
been 5 kmph, then she could swim 60 km:
A. Upstream in 4 hours

50
©Tathagat GS-Prep Ph: 8431824230/9113994724
ಗೆಲುವು GROUP C TEST SERIES

B. Downstream in 12 hours
C. Upstream in 6 hours
D. Downstream in 4 hours

Ans: B

Q85. Which of the following are correctly matched?


1. Lepakshi Temple – Hanging Pillar
2. Vittala Temple – Musical Pillars
3. Virupaksha Temple – 1000 Pillars
Choose the correct option
A. 1 and 3 only
B. 1 and 2 only
C. 2 and 3 only
D. 1, 2 and 3

Ans: A
Rudreshwara Swamy Temple is a historic Hindu temple located in the town of Hanamkonda,
Telangana which is known for 1000 Pillars. Pair 1st and 3rd are correctly matched.

Q86. Sariputta, Moggallanna are related to


A. Jainism
B. Ajivika
C. Buddhism
D. Shaivism

Ans: C
Sariputta and Moggallana (also called Maha Moggallana) were the two chief disciples of the
Buddha, often stylized as the right hand and left hand disciples of the Buddha respectively.

Q87. The places Guntapalle and Pitalkhora are famous for


A. Ancient temples
B. Archaeological excavations
C. Buddhist caves
D. Trade centres

Ans: C
The Guntupalle or Guntupalli Group of Buddhist Monuments is located near Kamavarapukota,
Eluru district, in the state of Andhra Pradesh in India. It is around 40 km away from Eluru. The
rock-cut part of the site has two Buddhist caves, a chaitya hall and a large group of stupas. The

51
©Tathagat GS-Prep Ph: 8431824230/9113994724
ಗೆಲುವು GROUP C TEST SERIES

chaitya hall has a rare carved stone entrance replicating wooden architecture, a simpler version of
that at the Lomas Rishi Cave. The Pitalkhora Caves, in the Satmala range of the Western Ghats of
Maharashtra, India, are an ancient Buddhist site consisting of 14 rock-cut cave monuments which
date back to the third century BCE, making them one of the earliest examples of rock-cut
architecture in India.

Q88. Which of the following Rulers was known as ‘Amitrochates/Amitragatha’ to Greeks which
means ‘slayer of enemies’.
A. Ashoka
B. Chandragupta Maurya
C. Dhanananda
D. Bindusara

Ans: D
Bindusara, also called Bindusara Maurya, Greek Amitrochates, (born c. 320 bce—died 272/3 bce),
second Mauryan emperor, who ascended the throne about 297 bce. Greek sources refer to him as
Amitrochates, Greek for the Sanskrit Amitraghata (“destroyer of foes”).

Q89. Consider the following statements regarding the state of women in the Sangam period.
1. Avvaiyar, Nachchellaiyar were major women poets of this period.
2. Karpu (chaste life) was considered as the highest virtue of women.
3. Sati was prevalent in the higher strata of the society.
4. Love marriage was a common practice.
Choose the correct statements.
A. 1, 2 and 4
B. 1, 2 and 3
C. 1 and 2 only
D. 1, 2, 3 and 4

Ans: D
While life of women in general was quite good, the condition of widows was miserable, hence
instances of practice of Sati have been recorded.

Q90. ‘TESE’, ‘GIFT’ and ‘MESA’ are the technologies related to which of the following?
A. Neutrino Detection
B. Machine Learning
C. Assisted Reproduction Technology
D. Blockchain Technology

Ans: C

52
©Tathagat GS-Prep Ph: 8431824230/9113994724
ಗೆಲುವು GROUP C TEST SERIES

Assisted Reproductive Technologies are a set of different ways to achieve pregnancy by keeping
sperm or oocyte outside the human body followed by transfer of gamete or embryo in the
reproductive tract of a woman. There are some technologies in ART services like Testicular Sperm
Extraction (TESE), Gamete Intrafallopian Transfer (GIFT), Microsurgical Epididymal Sperm
Aspiration (MESA), ovarian stimulation, Intrauterine Insemination (IUI), Zygote Intrafallopian
Transfer (ZIFT), Intracytoplasmic Sperm Injection (ICSI) etc.

Q91. Which of the following authorities is responsible for updating the list of critically
endangered, endangered and vulnerable species?
A. Botanical Survey of India
B. The National Board for Wildlife
C. Wildlife Protection Society of India
D. Zoological Survey of India

Ans: D
Zoological Survey of India
▪ The Zoological Survey of India (ZSI), a subordinate organization of the Ministry of
Environment and Forests was established in 1916.
▪ It is a national centre for faunistic survey and exploration of the resources leading to
the advancement of knowledge on the exceptionally rich faunal diversity of the
country.
▪ It has its headquarters at Kolkata and 16 regional stations located in different geographic
locations of the country.

Zoological Survey of India (ZSI) – Functions


The activities of the ZSI are coordinated by the Conservation and Survey Division under the
Ministry of Environment, Forest, and Climate Change. Its functions are as follows:
1. ZSI publishes the Red Data Book on Indian Animals. It was first published in 1983 and is
similar to the Red Data Book published by IUCN.
2. Publication of results including Fauna of India, Fauna of States, and Fauna of Conservation
Areas.
3. Training, Capacity Building, and Human Resource Development of the people involved.
4. Geographic Information System (GIS) and Remote Sensing studies on recorded animal
diversity as well as on threatened species.
5. It works for the development of Environmental Information System (ENVIS) and
Convention on International Trade in Endangered Species of Wild Fauna and Flora
(CITES) Centers.
• CITES is a multilateral treaty to protect endangered plants and animals.
6. Conducts collaborative research programs on “Biodiversity” with other organizations in
India and abroad.

53
©Tathagat GS-Prep Ph: 8431824230/9113994724
ಗೆಲುವು GROUP C TEST SERIES

Q92. Which of the following are freshwater lakes?


1. Pulicat lake
2. Wular lake
3. Loktak lake
Select the correct answer using the code given below:
A. 2 only
B. 2 and 3 only
C. 1 only
D. 1, 2 and 3

Ans: B
Pulicat lake is a saline and lagoon lake located at the border of Andhra Pradesh and Tamilnadu. It
is the 2nd largest brackish water lagoon in India after Chilika.

Q93. How many pairs are correctly matched:


1. Keibul Lamjao – Manipur
2. Dibru-Saikhowa – Assam
3. Dihang-Dibang – Arunachal Pradesh
Choose the correct answer from the code given below:
A. Only one pair
B. Only two pairs
C. None of the pairs
D. All three pairs

Ans: D
The three pairs mentioned in the question refer to national parks or wildlife sanctuaries located in
different states of India. The correct matches are as follows:
Keibul Lamjao – Manipur: This is a national park located in the Bishnupur district of Manipur. It
is the only floating national park in the world and is home to the endangered Manipur brow-
antlered deer, also known as the Sangai.
Dibru-Saikhowa – Assam: This is a national park located in the Tinsukia district of Assam. It is
known for its unique eco-system and is home to several rare and endangered species of flora and
fauna.
Dihang-Dibang – Arunachal Pradesh: This is a biosphere reserve located in the Upper Siang, West
Siang and Dibang Valley districts of Arunachal Pradesh. It is home to several rare and endangered
species of flora and fauna.

Q94. Consider the following statements about Ishwar Chandra Vidyasagar:

54
©Tathagat GS-Prep Ph: 8431824230/9113994724
ಗೆಲುವು GROUP C TEST SERIES

1. He started a movement in support of widow remarriage which resulted in legislation of


widow remarriage being passed by the British government.
2. He founded the Bethune School exclusively for providing English education to women.
Which of the statements given above is/are correct?
A. 1 only
B. 2 only
C. Both 1 and 2
D. Neither 1 nor 2

Ans: A
• In 1850, Ishwar Chandra Vidyasagar became the principal of Sanskrit College. He was
determined to break the priestly monopoly of scriptural knowledge, and for this he opened
the Sanskrit College to non-brahmins. He introduced Western thought in Sanskrit College
to break the self-imposed isolation of Sanskritic learning. As an academician, he evolved
a new methodology to teach Sanskrit. He also devised a new Bengali primer and evolved
a new prose style.
• Pt. Vidyasagar started a movement in support of widow remarriage which resulted
in legislation of widow remarriage.
• He was also a crusader against child marriage and polygamy. He did much for the cause of
women’s education. As government inspector of schools, he helped organise thirty-five
girls’ schools many of which he ran at his own expense. As secretary of Bethune School
(established in 1849), he was one of the pioneers of higher education for women in India.
• Bethune school was founded by John Elliot Drinkwater Bethune. (So, option 2 is
incorrect)

Q95. What was the objective of the Azamgarh Proclamation of 1857?


A. It offered a peace plan between zamindars and Indian rebels.
B. It laid down the manifesto for the rebels.
C. It served as an ultimatum to rebel sepoys to surrender.
D. It called upon the princely states to support the Indian rebels.

Ans: B
Azamgarh Proclamation (Azamgarh Ishtahār) was issued during the revolt of 1857. It was issued
(most probably) by Firoz Shah, a grandson of the Mughal Emperor who fought in Awadh, and
aims to set out a manifesto for what the rebels were fighting for.

Q96. In 1920, which of the following changed its name to “Swarajya Sabha”?
A. All India Home Rule League
B. Hindu Mahasabha
C. South Indian Liberal Federation

55
©Tathagat GS-Prep Ph: 8431824230/9113994724
ಗೆಲುವು GROUP C TEST SERIES

D. The Servants of India Society

Ans: A
“All India Home Rule League” is the name which was changed to “Swarajya Sabha” in 1920. The
“Swarajya Sabha” was under the leadership of the Freedom Fighters like Annie Besant and Bal
Gangadhar Tilak.

Q97. Arrange the following events in their correct chronological order:


1. Nehru Report
2. First Round Table Conference
3. First May Day in India
4. Dandi March
Select the correct code:
A. 3 1 4 2
B. 3 1 2 4
C. 1 3 4 2
D. 1 3 2 4

Ans: A
• In 1923, the first May day was celebrated in India in Madras.
• 1928 – Nehru Report
• Dandi March – March 12-April 6, 1930
• First Round Table Conference – November 1930 – January 1931

Q98. Whitley Commission established in British India was mandated to report on


A. Financial reforms needed in British administration
B. Police and military reforms needed in the wake of the 1857 mutiny
C. Trade and commerce within the princely states
D. Existing conditions of labour and to make recommendations

Ans: D
The Royal Commission on Labour or the Whitley Commission on Labour was set up in 1929
to inquire into the existing conditions of labour in industrial undertakings and plantations in India.
The Commission was chaired by John Henry Whitley. The commission submitted its report in
1931. The report surprised many by concurring with the criticisms of Mahatma Gandhi and others
that poverty was the cause of India’s social and industrial problems. It was also critical of British
employers’ role in perpetuating the problems.

Q99. Consider the following regarding the Revolt of 1857.


1. It was during the reign of Governor General Canning.

56
©Tathagat GS-Prep Ph: 8431824230/9113994724
ಗೆಲುವು GROUP C TEST SERIES

2. Mangal Pandey led the Mutiny.


3. It started from Delhi.
4. The mutiny did not spread to South India.
Which of the above statements is/are correct?
a. 1 and 2 only
b. 1 and 3 only
c. 2 and 3 only
d. 1 and 4 only

Ans: D
Statement 1 and 4 are correct. Statement 2, 3 are incorrect.
• The revolt of 1857 was held during the reign of Governor General Lord Canning.
(statement 1 is correct)
• The immediate cause of the revolt of 1857 was that Mangal Pandey killed two British
officers on parade at Barrackpore.
• This revolt spread in Lucknow, Ambala and Meerut cantonments. On May 10, 1857
soldiers at Meerut refused to touch the new Enfield rifle cartridges. The soldiers, along
with another group of civilians, went on rampage.
• They marched to Delhi, seized the city and proclaimed Bahadur Shah Zafar as the emperor
of India. Due to the lack of unity and organisation this revolt failed.

Q100. In context to Modern Indian history, what was the Mahad satyagraha about?
A. Protest against forced conversions into Hinduism in the guise of ‘Shudhikaran’
B. Challenge to the regressive customs of the caste Hindus
C. Indefinite strike against rise in local taxes from peasants
D. None of the above

Ans: B
Dr Bhimrao Ambedkar led the Mahad Satyagraha in March 1927 to challenge the regressive
customs of the caste Hindus. He stressed the necessity of removing ideas of ‘high’ and ‘low’ and
inculcating self-elevation through self-help, self-respect and self-knowledge. He led a procession
of some 2,500 ‘untouchables’ through the town of Mahad to the Chawdar tank, a public source of
water from which the untouchables were not allowed to draw water. Dr Ambedkar took water from
the tank and drank it. There were huge protests by caste Hindus.

57
©Tathagat GS-Prep Ph: 8431824230/9113994724

You might also like